Given the current economic climate, universal healthcare is an impossibility in the United States. More than half of

This topic has expert replies
Moderator
Posts: 7187
Joined: Thu Sep 07, 2017 4:43 pm
Followed by:23 members

Timer

00:00

Your Answer

A

B

C

D

E

Global Stats

Given the current economic climate, universal healthcare is an impossibility in the United States. More than half of all U.S. households report feeling overwhelmed by expenses, and many people are struggling to find additional sources of income. Funding such a massive program would require significant tax increases, adding to the financial burden of many individuals. The employer-sponsored healthcare system currently in place keeps taxes low, protecting our nation’s economy.

Which of the following, if true, would most weaken the argument above?

A. Many U.S. citizens enjoy sizeable tax breaks for medical expenses.

B. Universal healthcare would reduce the financial burden on employers, resulting in significant job growth and wage increases.

C. A majority of profitable, private health insurers have indicated that they expect to increase their payrolls in the coming quarter.

D. Pharmaceutical companies have fewer incentives to innovate new drugs in a universal healthcare system.

E. Most U.S. citizens depend on their employers for health coverage and could not afford comparable coverage under the current system.


OA B

Source: Princeton Review

Legendary Member
Posts: 2214
Joined: Fri Mar 02, 2018 2:22 pm
Followed by:5 members

Timer

00:00

Your Answer

A

B

C

D

E

Global Stats

What this argument says is that universal healthcare is impossible to achieve in the United States because it will require significant tax increases that will, in turn, increase the financial burden of individuals. On the other hand, the current employer sponsoring the system keeps taxes low.

So, in other to weaken the argument, you have to find the option that strengthens the point that universal healthcare will be financially beneficial.

Option A - incorrect
This is not true because the passage strengthens the current argument that the current employer sponsors the healthcare system.

Option B - correct
This weakens the argument because it supports the universal healthcare system instead of the current employers.

Option C - incorrect
It is wrong because it goes out of point. The passage argument doesn't talk about an increase in payrolls or the likes.

Option D - incorrect
This is also outside the scope of the argument. This is not concerned about pharmaceutical companies.

Option E - incorrect
This claim strengthens the argument because it doesn't address the benefits of the universal healthcare system.